Show that the nested intervals property fails for the rational numbers

Click For Summary
The discussion centers on demonstrating that the Nested Intervals property fails for the rational numbers by constructing a sequence of closed, bounded intervals whose intersection is empty. Participants suggest using the irrational number √2 to center these intervals, such as [√2 - 1/2, √2 + 1/2]. It is noted that these intervals do not contain their endpoints in the rational field, as both endpoints are irrational. The conversation raises questions about the topology assumed for the field of formal rational functions and clarifies the nature of intervals within this context. Ultimately, the discussion emphasizes the failure of the Nested Intervals property in the rational numbers.
Kb13
Messages
1
Reaction score
0
In the field of formal rational functions, construct a nest of closed, bounded intervals whose intersection is empty. (That is, show that the Nested Intervals property fails in this field)


I know it has to involve radical 2 but because that is the only number we know is irrational but other than that I have no idea.
 
Physics news on Phys.org
Try 'centering' your intervals around an irrational (you should know more irrationals than that, but sqrt2 is still a good choice). For example, one interval can be [sqrt2 - 1/2, sqrt2 + 1/2]. Turn this into a nested sequence and consider what happens as the length of the intervals get very small.
 
Kb13 said:
In the field of formal rational functions, construct a nest of closed, bounded intervals whose intersection is empty. (That is, show that the Nested Intervals property fails in this field)

Is there a topology assumed for this field? If so, what is it? I'm not sure what an "interval" looks like in this field.
 
I suspected he meant the set of rational numbers (since this is a common problem and he didn't know any other irrationals so odds are he doesn't know what a topology is).
 
Good point. I would like to point out one subtlety that he might otherwise overlook, namely that [sqrt2 - 1/2, sqrt2 + 1/2] in the rational field does NOT contain the endpoints since sqrt2 +/- 1/2 is irrational. It is nonetheless a closed (AND an open) interval.
 
Question: A clock's minute hand has length 4 and its hour hand has length 3. What is the distance between the tips at the moment when it is increasing most rapidly?(Putnam Exam Question) Answer: Making assumption that both the hands moves at constant angular velocities, the answer is ## \sqrt{7} .## But don't you think this assumption is somewhat doubtful and wrong?

Similar threads

Replies
5
Views
3K
  • · Replies 1 ·
Replies
1
Views
2K
  • · Replies 2 ·
Replies
2
Views
2K
  • · Replies 1 ·
Replies
1
Views
2K
  • · Replies 8 ·
Replies
8
Views
4K
Replies
3
Views
10K
  • · Replies 45 ·
2
Replies
45
Views
4K
  • · Replies 85 ·
3
Replies
85
Views
9K
Replies
32
Views
3K
  • · Replies 15 ·
Replies
15
Views
2K